Problem solving this volume using Jacobi's Determinant

Click For Summary
The discussion focuses on finding the volume of a solid defined by the equation 3x + 4y + 2z = 10 as the ceiling and several cylindrical surfaces as boundaries. The approach involves setting up a double integral to express the volume in terms of z, specifically ∫∫(3 - 1.5x - 0.5y) dx dy. However, the complexity of the limits prompts a suggestion to use a change of variables, which requires applying Jacobi's determinant. Visual aids are recommended to better understand the solid's shape and determine the integral limits. The conversation emphasizes the challenges of variable transformation in this context.
JorgeM
Messages
30
Reaction score
6

Homework Statement


Find the value of the solid's volume given by the ecuation 3x+4y+2z=10 as ceiling,and the cilindric surfaces
2x^2=y
x^2=3*y
4y^2=x
y^2=3x
and the xy plane as floor.

The Attempt at a Solution


I know that we have to give the ecuation this form:
∫∫z(x,y)dxdy= Volume
So, in fact we have to solve:
∫∫ ( 3 - 1.5x - .5y ) dxdy but actually it is easier to do variables' change because of the fact that the limits are to tricky to solve.

I get so confused when I try to suppose a good change and use in the Jacobi's determinant.
Hope you could help my because I got so confused :(

Thanks for your advise.
 
Physics news on Phys.org
Just to visualize your solid. On the (x,y) plane it looks like the grey area in

upload_2018-12-12_5-33-47.png
where the blue line is from the ceiling, a pyramid which crosses the z axis at 5 indicated by the pink area.

You can try to figure out your limits for the integrals with this picture in mind.
 

Attachments

  • upload_2018-12-12_5-33-47.png
    upload_2018-12-12_5-33-47.png
    4 KB · Views: 364
  • Like
Likes JorgeM
Question: A clock's minute hand has length 4 and its hour hand has length 3. What is the distance between the tips at the moment when it is increasing most rapidly?(Putnam Exam Question) Answer: Making assumption that both the hands moves at constant angular velocities, the answer is ## \sqrt{7} .## But don't you think this assumption is somewhat doubtful and wrong?

Similar threads

Replies
5
Views
2K
  • · Replies 9 ·
Replies
9
Views
2K
Replies
5
Views
1K
  • · Replies 10 ·
Replies
10
Views
2K
  • · Replies 8 ·
Replies
8
Views
2K
Replies
4
Views
1K
  • · Replies 13 ·
Replies
13
Views
3K
Replies
2
Views
2K
  • · Replies 6 ·
Replies
6
Views
2K
Replies
21
Views
2K